Elizabeth lives on a straight road that goes east and west. She starts from a point 5.8 miles west of her home and drives a certain distance to the store. The store is more than 434 miles east of her home.

Let d represent the distance Elizabeth drove.

Which inequality represents this situation?

Answers

Answer 1

Answer:

−5.8+d>43/4

Step-by-step explanation:

Answer 2

The inequality represents the situation as −5.8+d>434.

What is inequality?

When two expressions are connected by a sign like "not equal to," "greater than," or "less than," it is said to be inequitable. The inequality shows the greater than and less than relation between variables and the numbers.

Given that Elizabeth lives on a straight road that goes east and west. She starts from a point 5.8 miles west of her home and drives a certain distance to the store. The store is more than 434 miles east of her home.

The inequality will be written as the algebraic sum of the distance d and 5.8 is greater than the 434 miles.

Inequality: -5.8 > 434

To know more about inequality follow

https://brainly.com/question/24372553

#SPJ2


Related Questions

f(x)=−2x^2+2x−20
Find f(−8)

Answers

Answer:  -164

Step-by-step explanation:

Answer:-164

Step-by-step explanation:

−2(−8)

2

+2(−8)−20

Plug into each x-value

2

(

64

)

+

2

(

8

)

20

−2(64)+2(−8)−20

Square first

128

16

20

−128−16−20

Multiply

f

(

8

)

=

164

f(−8)=−164

Find the domain and range of this mapping diagram.

Answers

Answer:

Domain: 3,4,5,6

Range: 7,9,11,13

Pairs: (3,7), (4,9), (5,11), (6,13)

Step-by-step explanation:

En la estrella la

suma de los cuatro

números de la línea

es la misma. ¿Cuáles

son los valores de: b

y c?

Answers

Sabiendo que en la estrella la  suma de los cuatro  números de la línea  es la misma, la respuesta correcta es la opción c) los valores de b y c son 9 y 8 respectivamente.

En primer lugar, sabes que en la estrella la  suma de los cuatro  números de la línea  es la misma.

Por lo que es conveniente escoger aquella línea en donde todos los valores son conocidos, de manera de poder estimar el valor de la suma. De esta manera, se calcula:

12 +1 +2 +15= 30

Entonces la suma en todas las líneas debe dar como resultado 30.

Para calcular el valor de c y d, es preferible elegir aquellas líneas en donde solo aparezca b y donde solo aparezca c.

Entonces, escogiendo aquella línea donde se tiene como incógnita sólo el valor de b y sabiendo que debe sumar 30, se tiene:

12 + (8-3) + 4 + b= 30

Resolviendo:

12 + 5 + 4+ b= 30

21 + b=30

b= 30 - 21

b= 9

Por otro lado, eligiendo aquella línea donde se tiene como incógnita sólo el valor de c y sabiendo que debe sumar 30, se tiene:

15 + 3 +4 + c= 30

22 + c= 30

c=30 - 22

c= 8

Finalmente, se decide verificar en aquellas líneas donde aparecen ambos valores de c y d, o aquellas líneas que no se hayan utilizado previamente, de manera que sumen 30. Entonces:

Primera verificación: 2c +2 +3 +b= 2×8 +2 +3+ 9= 16 +2 +3+ 9= 30

Segunda verificación: 2c +1 +(8-3) +c= 2×8 +1 +5+ 8= 16 +1 +5+ 8= 30

De esta manera se comprueba que en todas las líneas de la estrella la suma de los cuatro números es la misma.

En resumen, la respuesta correcta es la opción c) los valores de b y c son 9 y 8 respectivamente.

Aprende más con este problema similar

https://brainly.com/question/25206811?referrer=searchResults

Jake fills a tank that can hold 200 gallons of water. The tank already has 50 gallons of water in it when Jake starts filling it at the rate of 10 gallons per minute. Karla fills a tank that can hold 300 gallons of water. That tank already has 100 gallons of water in it when Karla starts filling it at the rate of 5 gallons per minute. Jake and Karla start filling the tanks at the same time. How long after they start filling the tanks do the tanks have the same volume of water? What is that volume of water? I​

Answers

Answer:

Let m represent the number of minutes.

Amount of water in Karla's tank:  100 + 5m

Amount of water in Jake's tank:  50 + 10m

Same amount of water:  100 + 5m  =  50 + 10 m

--->                                         100  =  50 + 5m

--->                                            50  =  5m

--->                                            10 = m

In ten minutes, Karla's tank will have 100 + 5(10)  =  100 + 50  =  150 gallons.

In ten minutes, Jake's tank will have 50 + 10(10)  =  50 + 100  =  150 gallons.

Step-by-step explanation:

hope this helped!

if 18 plums weigh 54 ounces then 27 plums weigh x ounces

Answers

Answer:

81 ounces

Step-by-step explanation:

Divide 54 by 18 to find the weight of 1 plum then multiply this by 27.

54 ounces ÷ 18 = 3 ounces , then

27 plums = 27 × 3 = 81 ounces

Chen
An architect designs two similar triangular patos. The first patio has angle measures of (x-30), (y+15), and 60 The second patio has angle measures of X -10 40° and 80°

Answers

The values of x and y are 50 and 35 respectively.

The corresponding angles of similar triangles are equal. Therefore,

The first patio angles are:

x + 10

y + 15

70°

The second patio angles are ;

x + 20

50°

60°

Therefore,

x + 20 = 70

x = 70 - 20

x = 50

y + 15 = 50

y = 50 - 15

y = 35

can anyone please help me with this really quick

Answers

Answer:

90

Step-by-step explanation:

The sum of three numbers is equal to 185, so we can start with this equation, where x is the first number, y is the second, and z is the third:

x + y + z = 185

They tell us that y is 5 times greater than x, so we can insert that equation in for y in the original equation:

x + (5x) + z = 185.

And we are told z is 13 less than y, so we can insert that equation as well by using the equation before:

x + (5x) + (5x-13) = 185

So now we can combine like terms and solve for x.

6x + 5x - 13 = 185

11x - 13 = 185

11x = 198

x = 18

But we're not done, we still need to solve for the second number, which is y. So we insert the number for x into the original equation we have for y, which is that it's 5 times greater than x:

y = 5x

y = (5*18)

y = 90

Which expression is equivalent to

Answers

Step-by-step explanation:

(cubic root (x²))⁶ = (x^2/3)⁶ = x^2×6/3 = x^12/3 = x⁴

Evaluate the determinate:

I 5 3 3 I
I -4 -5 1 I
I 5 3 0 I
Please show work... I will give Brainliest.!

Answers

Answer:

39

Step-by-step explanation:

detailed explanation showed in the diagram above

please can someone help with this

Answers

Answer:

true

false

true

true

Step-by-step explanation:

A.

For lines p and q, angles 2 and 3 are corresponding angles, so A is true.

B.

Angles 5 and 7 hare note corresponding angles or alternate interior angles.  B is false.

C.

For lines r and s, angles 2 and 4 are corresponding angles, so C is true.

D.

For lines r and s, angles 5 and 6 are alternate interior angles, so D. is true.

What is the value of x?​

Answers

Answer:

Option D is correct.

Step-by-step explanation:

We can tell that

[tex]4x + \frac{x}{2} + 2 + \frac{3x-4}{2} = 180[/tex]

Now, let's simplify.

=> [tex]4x + 2 + \frac{4x-4}{2} = 180[/tex]

=> [tex]4x + 2 + 2x-2 = 180[/tex]

=> [tex]4x + 2x = 180[/tex]

=> [tex]6x = 180[/tex]

=> [tex]x = 30[/tex]

Therefore, Option D is correct.

Hoped this helped.

Part E
While diving, Winston stopped at a point located halfway between his deepest dive and the ocean’s surface. At what elevation was Winston when he stopped halfway? PLzzzzzz HURRRY UUUPPP Ill give whoever answers this RIGHT NOW 100pts, no joke

Answers

At the ocean’s surface, the elevation is 0. Meaning, his deepest dive divided by two would be halfway (and a negative)

Ex.
Deepest dive: -1000
Halfway: -500

Answer:

im pretty sure that the answer is -50.5 feet.

Step-by-step explanation:

mai has 36$ to spend on movie tickets each movie ticket cost 4.50$ how many ticket can she buy multiplication problem

Answers

Answer:

8

Step-by-step explanation:

Mai has $36 and each movie ticket is $4.50.

You wanna divide 36 by 4.50 to find out how much tickets she can purchase which is 8.

What is the measure of Segment AB if Segment BD is 10?*

Answers

Answer:

10

Step-by-step explanation:

all sides of square are the same

PLEASE HELP‼️‼️15 POINTS PLEASE ANWER ASAP‼️‼️‼️ILL MARK BRAINLYEST I MEAN IT ‼️‼️‼️
which theorem would be used to prove the triangles below as congruent?

Answers

Answer:

ASA

Step-by-step explanation:This is because the two triangles have two angles and a side that are equal to each other. pls brainlyest me

Adding this so you can give the other person a brainlyist

A principal of $2900 is invested at 4.25% interest, compounded annually. How much will the investment be worth after 7 years?

Answers

Let S = Sum after 13 years

     So = amount invested

     t = time in years

     i = annual interest rate = .0325

The S = So(1+i)t = $2,200(1.0325)13 = $3,334.21

Yesterday, jack drove 28 1/2 miles. He used 1 1/4 gallons of gasoline. What is the unite rate for miles per gallon?

Answers

Answer

MPG= 30 4/5 or 30.8 is I think is the answer.

9.43
9.43
15
15
16
© 2018 StrongMind. Created using GeoGebra.
What is the area, in square units of the composite figure?
Enter your answer as a number, like this: 42

Answers

Answer:

280

Step-by-step explanation:

first, find the area of the triangle

formula: 1/2 (base × height)

so the area of the triangle is

= 1/2 (16×5)

= 1/2 (80)

= 40

then, find the area of the rectangle

formula: length × breadth

the area of the rectangle is

= 15 × 16

= 240

the area of the composite figure is the sum of the areas of the triangle and rectangle

= 40 + 240

= 280

May I please receive help?

Answers

First, we know that a² + b² = c². We will just be using this in a different way! b² = c² - a²

b² = 32² - 28²

b² = 1024 - 784

b² = 240

b = 15.49

Hope this helps!

Hot Dog!
Moderate
Previous
If it costs a total of $9 to buy 1 hot dog and 2 bottles of water and it costs a total of $22 to buy 3 hot dogs and 4
bottles of water, how much does a single bottle of water cost?

Answers

Answer:

$2.50

Step-by-step explanation:

1h + 2w = 9

3h + 4w = 22

Multiply the first expression x 3

3h + 6w = 27

Subtract the 2 equations

3h + 6w = 27

3h + 4w = 22

2w = 5, w = 2.5

Determine which postulate or theorem can be used to prove that ASEA = A PEN E O A. ASA OB. SSS O C. AAS O D. SAS​

Answers

Answer:

asa is the correct answer

The postulate or theorem can be used to prove that triangles are congruent is ASA rule.

What are congruent triangles?

Two triangles are said to be congruent when they have congruent corresponding parts.

Given is a figure having two triangles, we need to find the rule, that can prove that both the triangles are congruent,

∠ S ≅ ∠ P [given]

∠ SEA ≅ ∠ PEN [vertically opposite angles]

SE ≅ EP [given]

∴ Δ SEA ≅ Δ PEN by ASA rule

ASA rule :-

If any two angles and the side included between the angles of one triangle are equivalent to the corresponding two angles and side included between the angles of the second triangle, then the two triangles are said to be congruent by ASA rule.

Hence, the postulate or theorem can be used to prove that triangles are congruent is ASA rule.

Learn more about congruent triangles click;

https://brainly.com/question/22062407

#SPJ7

How many 1/5 are in 3 1/2?

Answers

Answer:

17 1/2

Step-by-step explanation:

3 1/2 / 1/5=17 1/2

A 3-quart carton costs $2.79, while a 4-quart carton costs $3.88. Which is the better buy and why?

Answers

Answer: the 4 quart is a better buy

Step-by-step explanation:

Find the equation of the line passing through the points (-1,7) and
(2,-8). Write the equation in slope-intercept form.
A) y = -5x + 2
B) y = -5x + 7
C) y = 5x + 12
D) y = 5x – 18

Please help

Answers

A)
slop intercept form is y=mx+b

To find m, it is (y2-y1)/(x2-x1) or (-8-7)/(2-(-1)) which then = -5

To find b, the y-intercept choose one of the points and solve for b using:
y=x(slope)+b
7= -1( -5)+b
7= 5+b
2=b (subtract 5 on both sides)

I hope this helped!

One package of blueberries cost $3. How many packages of blueberries can you buy for $9?
Round the answer to the nearest whole number!

Answers

Answer:

3 packages

Step-by-step explanation:

To find how many packages you can buy for $9, you have to divide 9 by 3.

Since each package costs $3.

So, 9/3 = 3

So you can buy 3 packages of blueberries.

Answer:

3 Packages

Step-by-step explanation:

1 package costs $3

2 packages cost $6 (because 3+3=6)

3 packages cost $9- And we have exactly $9 so we can buy 3 packages.

what is the transformation of f(x)=|x-9|

Answers

Answer: X=9

Step-by-step explanation:

Answer:

f(x) = x-9 {9 < x}

f(x) = 0 {x = 9}

f(x) = -x - 9 {x < -9}

Step-by-step explanation:

I'm not too sure what transformation you want. However,,,

f(x)=|x-9| can be changed to function

f(x) = x-9 {9 < x}

f(x) = 0 {x = 9}

f(x) = -x - 9 {x < -9}

without absolute value.

help mee plss 5 mins

Answers

Answer:

20 ft

Step-by-step explanation:

It is 20 ft because you use pathagoren therom.

a^2 + b^2 = c^2

Answer:

The question cut off but I'm assuming this is basically just a find the area of the shaded region type of question. First thing you need to do is calculate the areas of the right triangle and the rectangle. Subtract the area of the triangle from the area of the rectangle.

Area of rectangle: 8 x 7 = 56ft^2

Area of the triangle: (6 x 2)/2 = 6ft^2

Subtract 6ft^2 from 56ft^2 and you get:

56 - 6 = 50ft^2

Area of shaded region is 50ft^2

The value of 63 is between what two numbers?
V
O 62 and 64
O 31 and 32
O 7 and 8
0 3 and 4

Answers

Answer:

62 and 64

Explanation:

As easy as counting,

62, 63, 64

Solve for x.
z=(x-8)k

Answers

Answer:

z/k + 8 = x

Step-by-step explanation:

z=(x-8)k

Divide both sides by k

z/k = x - 8

Add 8 to both sides

z/k + 8 = x

82. complete the check register for rob anderson. it has a previous balance of $175.40. He wrote check number 312 for $24.39 on November 22 to the barkber shop. The bank made an automatic transfer (AT) of $65.21 on November 25 to pay is cable bill. He made a deposit of $381.22 on November 27. What is the new balance in rob's account?

Answers

Answer:

$466.62

Step-by-step explanation:

Balance = $175 - $24.39 - $65.21 + $381.22

Use your calculator to add or subtract each new term as indicated:

$175 - $24.39 = $150.61

Next, subtract $65.21:  $150.61 - $65.21 = $85.40

Finally, add $381.22:  $85.40 + $381.22 = $466.62

on November 27 the new balance in rob's account became $466.62.

Other Questions
Chez soi! 1. Je ne mange pas chez What is the slope of the line tangent to the graph of y = (9x^2)/(x+2) at x = 1 ? Which expression is equivalent to 7 x + 6 ( x 8 2 ) 2 ( x + 3 ) + 10 ? A. 3 x + 40 B. 15 x 20 C. 3 x 20 D. 15 x + 40 Which of the following were leaders in the Cuban independence struggleHelp me please ASAP just number 1 Years ago, thousands of country music fans risked their lives by rushing to buy tickets for a Willie Nelson concert at Carnegie Hall. This behavior indicates Select one: a. nothing about the equilibrium price. b. the ticket price was at the equilibrium price. c. the ticket price was below the equilibrium price. d. the ticket price was above the equilibrium price. Her old eyes warmed at the sudden thought.This suggests that Correct the text by inserting one comma.Margaret Thatcher was the United Kingdom's first female prime minister and she proved to be one of the most influential leaders of the twentieth century the formula to find the curved surface area of the cone People of color were required to live in regulated areas.What were the regulated areas called? In 1 paragraph of sentences, whats something you've learned about the effects of alcohol? Which is 5log x - 6log (x - 8) written as a single logarithm? Which two events disrupted overland trade between Europe and Asia please help asap!!!!!!!!!!! Is this correct? And if it is, what's the solution? What is the best inference that can be made about the author's message inparagraph 17? 1) What is the BEST way to summarize paragraph 5? Anyone a big fan of 1D?? let f(x)=5x+5 find f(-1)I need your help with this. please help me Give me the answer please a big bag contains 500 beads, each of the same size, but in 5 different colors. Suppose there are 100 beads of each color and I am blindfolded. What is the fewest number of beads I must pick to be absolutely sure there are 5 beads of the same color among the beads I have picked blindfolded?I WILL REPORT ANY FAKE OR USELESS ANSWERS